Difference between revisions of "1997 JBMO Problems/Problem 4"
Rockmanex3 (talk | contribs) m (→Solution) |
Rockmanex3 (talk | contribs) m (Conciseness change (thanks Quantomaticguy)) |
||
(One intermediate revision by one other user not shown) | |||
Line 3: | Line 3: | ||
Determine the triangle with sides <math>a,b,c</math> and circumradius <math>R</math> for which <math>R(b+c) = a\sqrt{bc}</math>. | Determine the triangle with sides <math>a,b,c</math> and circumradius <math>R</math> for which <math>R(b+c) = a\sqrt{bc}</math>. | ||
− | == Solution == | + | == Solutions == |
+ | |||
+ | ===Solution 1=== | ||
Solving for <math>R</math> yields <math>R = \tfrac{a\sqrt{bc}}{b+c}</math>. We can substitute <math>R</math> into the area formula <math>A = \tfrac{abc}{4R}</math> to get | Solving for <math>R</math> yields <math>R = \tfrac{a\sqrt{bc}}{b+c}</math>. We can substitute <math>R</math> into the area formula <math>A = \tfrac{abc}{4R}</math> to get | ||
Line 21: | Line 23: | ||
Note that <math>2\sqrt{bc}</math>, so multiplying both sides by that value would not change the inequality sign. This means | Note that <math>2\sqrt{bc}</math>, so multiplying both sides by that value would not change the inequality sign. This means | ||
<cmath>0 < b+c \le 2\sqrt{bc}.</cmath> | <cmath>0 < b+c \le 2\sqrt{bc}.</cmath> | ||
− | + | However, by the [[AM-GM Inequality]], <math>b+c \ge 2\sqrt{bc}</math>. Thus, the equality case must hold, so <math>b = c</math> where <math>b, c > 0</math>. When plugging <math>b = c</math>, the inequality holds, so the value <math>b=c</math> truly satisfies all conditions. | |
− | |||
− | |||
− | |||
− | |||
<br> | <br> |
Latest revision as of 14:48, 23 February 2021
Contents
Problem
Determine the triangle with sides and circumradius for which .
Solutions
Solution 1
Solving for yields . We can substitute into the area formula to get We also know that , where is the angle between sides and Substituting this yields Since is inside a triangle, . Substitution yields Note that , so multiplying both sides by that value would not change the inequality sign. This means However, by the AM-GM Inequality, . Thus, the equality case must hold, so where . When plugging , the inequality holds, so the value truly satisfies all conditions.
That means so That means the only truangle that satisfies all the conditions is a 45-45-90 triangle where is the longest side. In other words, for all positive
See Also
1997 JBMO (Problems • Resources) | ||
Preceded by Problem 3 |
Followed by Problem 5 | |
1 • 2 • 3 • 4 • 5 | ||
All JBMO Problems and Solutions |